Proving Boundedness of Symmetric Operator on Hilbert Space

Click For Summary
SUMMARY

The discussion centers on proving the boundedness of a symmetric operator A on a Hilbert space X, where D(A) = X and (Ax, y) = (x, Ay) for all x, y in H. It is established that if A is symmetric and defined everywhere, it is self-adjoint and closed, leading to the conclusion that A is continuous and thus bounded. The Hellinger-Toeplitz theorem is referenced, confirming that unbounded symmetric operators cannot be defined on the entire Hilbert space.

PREREQUISITES
  • Understanding of Hilbert spaces and their properties
  • Knowledge of symmetric and self-adjoint operators
  • Familiarity with the closed graph theorem
  • Concept of operator continuity in functional analysis
NEXT STEPS
  • Study the Hellinger-Toeplitz theorem in detail
  • Learn about the closed graph theorem and its implications for operators
  • Explore the properties of self-adjoint operators in Hilbert spaces
  • Investigate the relationship between boundedness and continuity of linear operators
USEFUL FOR

Mathematicians, graduate students in functional analysis, and anyone studying operator theory in Hilbert spaces will benefit from this discussion.

Pietjuh
Messages
75
Reaction score
0

Homework Statement


Let A be a linear operator on a Hilbert space X. Suppose that D(A) = X,
and that (Ax, y) = (x, Ay) for all x, y in H. Show that A is bounded.

The Attempt at a Solution



I've tried to prove it by using the fact that if A is continuous at
a point x implies that A is bounded.

Suppose that x_n converges to x.
|| Ax_n - Ax ||^2 = || A(x_n - x) ||^2 = ( A(x_n - x), A(x_n - x) ) =
(x_n - x, A^2 (x_n - x) ) <= ||x_n - x || || A^2(x_n - x) ||

But i don't think I can conclude from this that because ||x_n - x|| -> 0, this expression goes to zero, since || A^2(x_n - x) || may blow up.. Or doesn't it?

Please help me :)
 
Physics news on Phys.org
D(A)=domain of A? I don't see how you can prove this. Self adjointness doesn't imply boundedness. You must be assuming A is also continuous? Then there is not much to prove. Better check the question.
 
Last edited:
Pietjuh said:

Homework Statement


Let A be a linear operator on a Hilbert space X. Suppose that D(A) = X,
and that (Ax, y) = (x, Ay) for all x, y in H. Show that A is bounded.

In a Hilbert space bounded <=> continuous. If A is symmetric and everywhere defined, it's equal to its adjoint, hence self-adjoint hence closed. By the closed graph theorem, it is also continuous, end proof.

The theorem you needed to prove is called "The Hellinger-Toeplitz theorem" and proves that unbounded symmetric operators cannot be defined on all \mathcal{H}.
 
Question: A clock's minute hand has length 4 and its hour hand has length 3. What is the distance between the tips at the moment when it is increasing most rapidly?(Putnam Exam Question) Answer: Making assumption that both the hands moves at constant angular velocities, the answer is ## \sqrt{7} .## But don't you think this assumption is somewhat doubtful and wrong?

Similar threads

Replies
1
Views
1K
  • · Replies 9 ·
Replies
9
Views
2K
  • · Replies 3 ·
Replies
3
Views
2K
  • · Replies 15 ·
Replies
15
Views
2K
  • · Replies 2 ·
Replies
2
Views
2K
  • · Replies 1 ·
Replies
1
Views
2K
  • · Replies 7 ·
Replies
7
Views
2K
  • · Replies 13 ·
Replies
13
Views
3K
  • · Replies 2 ·
Replies
2
Views
4K
  • · Replies 43 ·
2
Replies
43
Views
4K